5 votos

Cálculo del valor de expectativa del vacío en la Teoría de Perturbaciones Quiral

Desde un punto de vista tengo que admitir desde el principio que esta pregunta podría ser un problema de cálculo trivial pero la verdad es que estoy bastante atascado. Estoy estudiando la teoría de perturbación quiral con el objetivo de construir el $\chi PT$ Lagrangiano. Por esta razón examino primero la aparición del condensado de quarks.

Más detalladamente, tenemos la densidad pseudoescalar $P_i= \bar q \gamma_5 \tau_i q$ y estamos trabajando sólo con los quarks up y down, por lo que la simetría completa del Lagrangiano QCD es $SU(2) \times SU(2) $ .

Mi problema es cómo calcular el VEV de P. Es decir, el hecho de que las matrices de Pauli $\tau_i$ son $2 \times 2$ dimensional y el $\gamma_5$ es $4 \times 4$ . Trabajo en la base de Dirac para la $\gamma_5$ para que sea cero en la diagonal y $I_{2 \times 2} $ en las otras entradas. No sé si debo utilizar de alguna manera el hecho de que la u y la d son espinores de Dirac o, como encuentro en la página 81 de https://arxiv.org/abs/hep-ph/0210398 utilizar una relación similar a la relación 4.17 - que de hecho no entiendo.


EDITAR

Para ponerlo en una imagen más matemática, lo que deseamos calcular es algo de la forma $$\bar q \gamma_5 \tau_3 q $$ donde q es $$q= \left( \begin{array}{c} {u \\ d } \end{array} \right) = \left( \begin{array}{c} u_1 \\ u_2 \\ u_3 \\ u_4 \\ d_1 \\ d_2 \\ d_3 \\ d_4 \end{array} \right),$$ es decir, tanto u como d son espinores de Dirac. Pero tengo en mente que la matriz gamma es $4 \times 4 $ y la matriz de Pauli es $2 \times 2 $ lo que me parece bien porque sólo actúa sobre las componentes u y d de isospín.

Entonces, ¿cómo debo realizar la acción en el $\gamma_5 $ matriz en el $q$ que en realidad es un espinor de Dirac de dos componentes?

Gracias, señor.

0 votos

Ummm... los Ps son campos interpolantes QCD para los mesones pseudoescalares, los piones, mientras que los S para los mesones escalares... los sigma.... así que (4.17-18) conectan los Ps con los Ss a través de la acción de una carga axial, que no aniquila el vacío.... Es el s.a.r. de (4.17-18) el que tiene un v.e.v. no trivial, el condensado quiral, y no lo calculas, a no ser que hagas teoría gauge de celosía. La tabla de p 81 te dice lo que es, y lo correlacionas con propiedades de las pseudoescalares...

2voto

thierryb Puntos 1269

Como he indicado, no deseo hacer muecas a su artículo de revisión, pero el texto de TP Chang y LF Li cap. 5.4, 5.5, es menos antipático.

En cualquier caso, usted parece perdido, y un mapa del terreno es en orden así que seré esquemático, ya que todas las fórmulas correctas están ahí, pero su conectividad parece perdida... Todo índices matriciales en el producto directo $\gamma \otimes \tau_a$ están saturados por espinores, por lo que sólo se trata de combinaciones escalares y pseudoescalares aromatizadas. S a , P a y sin sabor: S, P . (Por lo tanto, para responder a su edición de lado-pregunta, el $\gamma_5$ acción es diagonal de sabor: conecta u s con u s y d s con d s.)

La cuestión es que la QCD no-perturbativa produce un condensado escalar, es decir, sólo los Ss recogen un v.e.v. y nunca los Ps la última línea de la tabla 4.1, p. 81, por lo que $\langle S\rangle = \langle \bar{q} q\rangle \neq 0$ lo que la gente normal llama el .

Así que, para empezar, te estás preguntando cómo calcular un 0, que en realidad es un dato. La QCD, como teoría vectorial, no puede romper la paridad. Para empezar, tampoco puede romper el isospín (que se rompe explícitamente por las pequeñas masas de los quarks), así que relaciona las componentes vectoriales de $\langle S_a\rangle=0$ para producir restricciones adicionales en bilineales, como lo hace en (4.15), para isospin sólo $\langle \bar{u} u \rangle = \langle \bar{d} d \rangle $ .

No estás calculando el v.e.v. de piones, sino, efectivamente, de bilineales de tales objetos (cada uno un bilineal de quarks). Sin sumar sobre índices de isospín repetidos, (4.17,8), $\langle \delta_a P_a\rangle\neq 0$ se obtiene la versión no evanescente no integrada de (4.19). La carga axial no puede aniquilar el vacío, ni tampoco la densidad pseudoescalar P a .

Ahora, las bilineales pseudoescalares $P_a$ son los campos interpolantes QCD de los 3 piones, los campos Goldstone de esta ruptura espontánea/dinámica de la simetría quiral, y definitivamente no tienen v.e.v.s no evanescentes--recuerde la quiral modelo en secciones anteriores, aunque no aniquilen el vacío, por encima.

Pero ellos do saber sobre el vacío... En lugar de matarlo, excluido anteriormente, se deslizan dentro y fuera de él, a través de la ecuación más importante del artículo--y de cualquier artículo sobre el tema--(4.19), PCAC, la madre de todos los teoremas de piones blandos en álgebra actual , $$ \langle 0| A_\mu^a (0) |\pi^b (p)\rangle= ip_\mu F \delta _{ab}. $$

El campo axial del centro es la corriente de carga quiral, por lo que, en transformada de Fourier, su componente 0 integrada sobre el espacio daría 0 actuando a la izquierda, Si el vacío fuera invariante quiralmente ¡pero no lo es! (En realidad, la carga quiral tiene problemas infrarrojos en su definición, según la Teorema de Fabry-Picasso pero no nos preocupemos...) Lo que hace es generar un pión en esta realización no lineal, que aniquila al pión sobre el que actúa, a la derecha.

Así, tomando la divergencia de esta corriente en el espacio de Fourier, obtenemos $m_\pi^2$ en el lado derecho, y que debería desaparecer en un mundo ideal (CAC) en el que $\partial^\mu A^a_\mu=0$ . Pero este es el sucio mundo real en el que la simetría quiral es aproximada hasta las masas de los quarks, por lo que la masa del pión puede sobrevivir para ser distinta de cero (P de "parcialmente") siempre que haya pequeñas masas de quarks "semilla" que impidan que sea un goldston, como requiere el teorema de Goldstone: Los piones son pseudogoldston.

Su v.e.v. se desvanece en un vacío único, |0>, pero la carga quiral se atasca en el vacío y crea estados de frecuencia casi (pseudo) cero de ellos.

0 votos

Gracias, profesor. Me preguntaba si también podría hacerme un comentario sobre esto: physics.stackexchange.com/questions/355827/ . Pero, de todos modos, su consideración ha sido de lo más útil.

i-Ciencias.com

I-Ciencias es una comunidad de estudiantes y amantes de la ciencia en la que puedes resolver tus problemas y dudas.
Puedes consultar las preguntas de otros usuarios, hacer tus propias preguntas o resolver las de los demás.

Powered by:

X